Please help me ( This the the owner of this acc sister again) Please don't remove this!

Please Help Me ( This The The Owner Of This Acc Sister Again) Please Don't Remove This!

Answers

Answer 1

Answer:

Sam won first, Craig won second, Randy won third, and Charlie won fourth.

Step-by-step explanation:

Sam won first, Craig won second, Randy won third, and Charlie won fourth. It's all a matter of which number is bigger, which means it took more time.


Related Questions

Amanda is running a mile in gym class. She runs at a pace of 5 minutes per mile. What's the dependent variable in the situation?

Answers

The dependent variable should be the pace Amanda is running since it determines how many minutes it takes her to run a mile. (so sorry if it's wrong!)

PLZ HURRY ILL GIVE BRAINLIEST
Simplify. 6(5p^3 − 6p)
ALSO, HOW WAS YOUR DAY?

Answers

Answer:

p = {0, -1.095445115, 1.095445115}Step-by-step explanation:

Answer:

30p^3-36p. Also, my day was niceeeee.

Step-by-step explanation:

To simplify this, we have to use the distributive property. First, you do 6 times 5p^3, and then you get 30p^3. Then you do 6 times 6p, which gives you 36p. So, the final answer would be 30p^3-36p.

Diana works in a factory. She receives a salary of $8 per hour and piecework pay of 12
cents per unit produced. Last week she worked 38 hours and produced 755 units.
a. What was her piecework pay?
b. What was her total hourly pay for the week?
c. What was her total pay for the week?
d. What would her total weekly salary have been if she produced 0 units?

Answers

Answer:

A) $90.60

B)$304

C)$394.60

D)$304

Step-by-step explanation:A) If she makes 8 cents per product produced and she's produced 755, you multiply 755 times 8 and get 6,040. Place the tenths decimal and you get your answer.

B)She makes 8 an hour and works 38 hours, 38 multiplied by 8 equals $304.

C) Add the answers to both A and C and you get $394.6

D) Refer back to B

Help!!due today help as quickly plz​

Answers

Answer:

Step-by-step explanation:

The answer will be

2/16 + 5/8 = 3/4

 


The equation of a circle whose center is at (4, 0) and radius is length 2√(3) is

(x - 4)² + y² = 2√3
(x - 4)² + y² = 12
(x + 4)² + y² = 12

Answers

Answer:

B. (x - 4)² + y² = 12

Step-by-step explanation:

Rewrite in standard form to find the center (h,k) and radius r.

Answer:

B?

Step-by-step explanation:

If the height is changed to 3, does this have the same effect on the volume as changing the radius to 1?


If the height is now 3, then the new volume is π units3.


If the radius is now 1, then the new volume is π units3.


Therefore, changing the height to half of its original value and changing the radius to half of its original value does the volume. Halving the height of the cone the volume, while halving the radius of the cone results in the volume.

Answers

Answer:

4, 2, not have the same effect on, halves, 1/4

Step-by-step explanation:

pleeeeeez mark brainliest

Halving the height of the cone halves the volume, while halving the radius of the cone results in 1/4 the volume.

How to find the volume of a cone?

We are given a right cone with;

radius = 2

height 6

volume is; V = ¹/₃π(2)²6 = 8π units³

If the height is now 3, then the new volume is;

¹/₃π(2)²3 = 4π units.

If the radius is now 1, then the new volume is;

¹/₃π(1)²6 = 2 units.

Therefore, changing the height to half of its original value and changing the radius to half of its original value does not have the same effect on the volume. Halving the height of the cone halves the volume, while halving the radius of the cone results in 1/4 the volume.

Read more about Volume of Cone at; https://brainly.com/question/12004994

Find the midpoint of the line segment with end coordinates of: ( 1 , 7 ) and ( 3 , − 2 ) Give coordinates as decimals where appropriate.

Answers

Answer:

(2, 2.5)

Step-by-step explanation:

To find the midpoint, the middle point of a line segment

Add the x coordinates together and divide by 2

(1+3)/2 = 4/2 =2

Add the y coordinates together and divide by 2

(7-2)/2 = 5/2 =2.5

___________________________________

The right answer is (2,2.5)

please see theattachedpicture for full solution

Hope it helps

Good luck on your assignment

Stay safe...

__________________________________

Solve by taking the square root of both sides.
3 (x +3)^2 – 81 = 0
a. X = -3 -3 sqrt 3 or -3 + 3 sqrt 3 c. 3 sqrt 3 or 3 + 3 sqrt 3
b. X = -3 -6 sqrt 3 or -3 + 6 sqrt 3 d. 3 sqrt 3 or -3 + 3 sqrt 3

Answers

Answer:

a

Step-by-step explanation:

Given

3(x + 3)² - 81 = 0 ( add 81 to both sides )

3(x + 3)² = 81 ( divide both sides by 3 )

(x + 3)² = 27 ( take the square root of both sides )

x + 3 = ± [tex]\sqrt{27}[/tex] = ± 3[tex]\sqrt{3}[/tex] ( subtract 3 from both sides )

x = - 3 ± 3[tex]\sqrt{3}[/tex]

x = - 3 - 3[tex]\sqrt{3}[/tex] or x = - 3 + 3[tex]\sqrt{3}[/tex] → a

Answer:

a

Step-by-step explanation:

Angles L and M are supplementary. If m ∠ L = (x+2)° and m ∠ M = (x-6)°. What is the value of x?

Answers

The value of X is 92.

please see the attached picture for full solution

Hope it helps

If using the method of completing the square to solve the quadratic equation
x2 + 8x – 4 = 0, which number would have to be added to "complete the square"?

Answers

Add 20 to complete the square

Can someone help me with 5?

Answers

Answer:

C

Step-by-step explanation:

"at least" is denoted by ≥. The amount of figures Carlos and Josh have can be represented by 5x + 4 and 3x + 14 respectively where x is the amount of figures in a set. We can write:

5x + 4 ≥ 3x + 14

2x ≥ 10

x ≥ 5

the answer is c

d
eddjiejjeihrhe

What is the name of the relationship between ∠1 and ∠4?



alternate exterior angles


adjacent angles


corresponding angles


alternate interior angles

Answers

Answer:

they are corresponding angles

Step-by-step explanation:

Overestimate the grocery bill with items that cost: $8.56, $5.34, $34.25, and $6.80.

A. $55.00

B. $57.00

Answers

Answer:

B. $57.00

Step-by-step explanation:

The totals add up to just under $55.00, so an overestimate would be $57.00


In a group of 60 students, 18 students take Algebra I, 20 students take
Algebra II, and 10 students take both subjects. How many students don't take
either of these subjects?

A. 34
B. 36
C. 32
D. 12

Answers

The Answer would be D. 12

Answer

D (12)

Step-by-step explanation:

60-20=40

40-18=22

22-10=12

A tree outside Ellie’s house cast of 125 foot shadow. At the same time of day, Ellie cast a 5.5 foot shadow. If Ellie is 4‘10“ tall, how tall is the tree in feet?

Answers

Answer:

The tall of the tree is about 109.85 feet

Step-by-step explanation:

- The tree and its shadow formed a right angle triangle with legs

 x the tall of the tree and 125 feet the shadow of the tree

- Ellie and her shadow formed a right triangle with legs 4 feet and

10 inches the tall of Ellie and 5.5 feet the shadow of Ellie

- The two triangles are similar

- There is an equal ratio between the corresponding sides of the

similar triangles

∵ The tall of the tree is x

∵ The tall of Ellie is 4 feet and 10 inches

babyface17

∴ The tall of Ellie is 4 feet and 5/6 foot = 4 + 5/6 = 29/6 feet

∵ The shadow of the tree is 125 feet

∵ The shadow of Ellie is 5.5 feet

- By using similarity ratio

∴ Tall of tree/tall of Ellie = shadow of tree/shadow of Ellie

∴ x/(29/6) = 125/5.5 ⇒ using cross multiplication

∴ 5.5(x) = 125(29/6) ⇒ divide both sides by 5.5

∴ x ≅ 109.85 feet

* The tall of the tree is about 109.85 feet

Answer:

109.85 ft

Step-by-step explanation:

5.5'/ 4'10"=125 ft/x

5.5*12/(4*12+10)=125/x

66/58=125/x

x=125*58/66

x=109.85 ft

I WILL MAKE BRAINLIEST AND 30pts. FAST AND HELP ME

Answers

Answer:

3rd,5th,6th

Step-by-step explanation:

complementary angles has 90 degree angles and 5x+40 = 5x10+40 = 50+40 = 90 and adjacent angles are angles that add up to a complementary or supplementary angle and do not overlap.

Hope this helps!

Answer:

Step-by-step explanation:

3rd option

x=10

angle ABD + angle DBC = 90

ie 5x + 40= 90

5x=90-40

5x=50

x=10

5th option

both are complementary angles

as  they are sum is 90 degree

hope this helps you and dont forget to make me as the brainliest......

which inequality matches the graph?

Answers

the fourth one i believe

If $3,000 is deposited at the end of every six months into an account that earns 10% semiannually, find the amount after 10 years.

Answers

Answer:

7959.83

Step-by-step explanation:

A = P ( 1+r/n)^ nt

Where A is the amount in the account

P is the principal

r is the rate

n is the number of compoundings per year

t is the years

A =3000 ( 1+.10/2)^ 2*10

A =3000 ( 1+.05)^ 20

   =7959.83

Answer:

$66,000

Step-by-step explanation:

if they earn 6,000 every year and  another $600 through interest, add 6,000 + 600 = 6,600 x 10 = $66,000.

Is 7 a factor of 56w - 7?​

Answers

Answer:

yes it is

Step-by-step explanation:

Factor, in mathematics, a number or algebraic expression that divides another number or expression evenly—i.e., with no remainder. For example, 3 and 6 are factors of 12 because 12 ÷ 3 = 4 exactly and 12 ÷ 6 = 2 exactly. ... The prime factors of a number or an algebraic expression are those factors that are prime.

Numbers we can multiply together to get another number.

Example: 2 and 3 are factors of 6, because 2 × 3 = 6

A number can have MANY factors!

Example: What are the factors of 12?

• 3 × 4 = 12, so 3 and 4 are factors of 12

• 2 × 6 = 12, so 2 and 6 are also factors of 12

• and 1 × 12 = 12, so 1 and 12 are factors of 12 as well

So 1, 2, 3, 4, 6 and 12 are all factors of 12

And -1, -2, -3, -4, -6 and -12 also, because multiplying negatives makes a positive.

In Algebra, factors can be expressions like "x+3" etc

please mark brainliest :)

Answer:

56w-7 56/7= 8 -7/7=-1

Step-by-step explanation:

8w-1 answer here

A bag contains 6 red tiles and 5 yellow tiles. Another bag contains 8 red tiles and 2 yellow tiles. A token is removed from each bag. What is the probability that both tiles are yellow?

Answers

Answer:

The probability is most likely to occur

Step-by-step explanation:

answer both, please. and explain

Answers

Answer:

4. [tex]\frac{13}{23}[/tex]

5. [tex]\frac{4}{7}[/tex]

Step-by-step explanation:

4. The relative frequency is how often it happens divided by the whole number of outcomes.

In this case it would be [tex]\frac{13}{23}[/tex]

5. Find the relative frequency again:

[tex]\frac{12}{28}=\frac{4}{7}[/tex]

1) Lakisha wants to buy some bitcoins. The exchange rate is $1 USD to 0.003 bitcoin. How many bitcoins can she buy with $400?

Enter your answer as a decimal, for example: 4.6

Answers

Answer:

1.2

Step-by-step explanation:

Calculate the sum of the first 50 terms in the sequence.
18, 21, 24, 27, ...

Answers

Answer:

hope this is correct

The sum of the first 50 terms in the sequence 18, 21, 24, 27, ... is 4575.

What is the sum of the sequence?

The sum of the sequence can be calculated by the formula

[tex]S_n = n/2(2a +(n-1)d)[/tex]

where a is the first term, d is the common difference

To find the sum of the first 50 terms in the sequences: 18, 21, 24, 27, ...

[tex]S_n = n/2(2a +(n-1)d)[/tex]

here, a = 18

d = 21 - 18 = 3

n = 50

[tex]S_{50} = 50/2(2(18) +(50-1)3)\\S_{50} = 25(36+147)\\S_{50} = 4575[/tex]

Thus, the sum of the first 50 terms in the sequence is 4575.

Learn more about sequence ;

https://brainly.com/question/21961097

What is the measure of DAB ?

Answers

Answer: I think it might be the angle in between D and B

Step-by-step explanation: My Explanation to this is:All triangles add to 180° so...

Add all the angles in the triangle.

Find the amount missing to make an overall of 180 degrees.

85+33=118 degrees

We are currently missing angle which is 62 degrees.

How ever to find the second missing angle you will need this angle (62) specifically.

because it is on a straight line, take 62 away from 180 to get 118 degrees

the answer is 118 degrees hopefully this helped you

Answer:

Step-by-step explanation:

Exterior angle equals the sum of opposite interior angles

∠DAB = ∠ABC + ∠ACB

          = 85 + 33

∠DAB = 118°

Which statement is true about the green,orange,and gray triangles show in the textile pattern below?

A. They are congruent obtuse triangle
B. They are congruent right triangle
C. They are acute right triangle
D. They are similar right triangle

Answers

Answer:

b

Step-by-step explanation:

the triangles are all the same so they are congruent. They are also right triangles so they must be a right triangle the only answer that combine this is b

The green, orange and gray triangles are congruent and right triangles. Thus option B is correct.

What are congruent triangles and right triangle?

Two triangles are congruent triangles if they are of same size and shape.

Right triangle is a triangle with one of angle 90°.

The given triangles of green, orange and gray triangles are of same shape and size . Therefore  we can say that they are congruent triangles

Also we can see that they all have right angle on the left corner therefore they are right triangles . If they are right triangles then they will be also acute triangles too as the some of other two angles of each triangle will be 90 it means they both are less than 90 and congruent means they are similar too as similar triangles have same shape.

Therefore the most appropriate option is Option B , that given triangles are congruent right triangles.

Also, Learn more about the congruent triangles from the given below link:

https://brainly.com/question/22062407

#SPJ5

Find the slope of the line that passes through the points A(-6, 2) and B(5, -1).

Answers

Answer:

-3/11

Step-by-step explanation:

The slope of the line is given by

m = (y2-y1)/(x2-x1)

   = (-1-2)/(5--6)

   = -3/(5+6)

   = -3/11

Answer:

[tex]\frac{-3}{11}[/tex]

Step-by-step explanation:

You use the formula to solve:

[tex]m = \frac{y_2-y_1}{x_2-x_1}[/tex]

[tex]m =\frac{-1-2}{5+6}\\m = \frac{-3}{11}[/tex]

Line CD contains points A (4, −7) and B (4, 8). The slope of line CD is

Answers

Answer:

Undefined

Step-by-step explanation:

Slope of CD

[tex] = \frac{8 - ( - 7)}{4 - 4} = \frac{8 + 7}{0} = \infty \\ [/tex]

Hence, Slope of CD is undefined.

Which matrix represents the system of equations shown below?
8x-3y = 9
2x + 5y = -7

Answers

Answer:

The correct answer is D.

Step-by-step explanation:

hope it works out !!

Answer:

its A

Step-by-step explanation:

i just did it

Circular lawn has a row of bricks around the edge the radius of the lawn is about 20 feet which is the best estimate for the area of the grass in the long use 3.14 for pie select one 125 ft 125 square ft 1250 ft 1256 square feet

Answers

Answer:

Area of the grass in the circular lawn is 1256 square feet

Step-by-step explanation:

Lawn is in the shape of circle having radius = 20 feet

Area of the circular lawn will be represented by the formula,

Area = πr² [ Here 'r' = Radius of the circular lawn]

        = (3.14)(20)²

        = 3.14 × 400

        = 1256 square feet

Therefore, area of the grass in the circular lawn will be 1256 square feet.

If a point Cis inside ZAVB, then mŁAVC+
= MZAVB.
m ZAVB = 62°
A. MZCVB
B. MBVA
C. MZAVB
o
D. MZAVC
SUBMI
- PREVIOUS

Answers

Answer: A

Step-by-step explanation:

The missing angle is <CVB.

What is an Angle?

An angle is formed by joining two rays (half-lines) that have a shared terminal. The latter is referred to as the angle's vertex, while the rays are referred to as the angle's sides, legs, and arms.

An angle is a figure in Plane Geometry generated by two rays or lines that share a common endpoint. The term "angle" is derived from the Latin word "angulus," which meaning "corner".

We have m < AVC = 39

and, m <VCB = 23

Thus, by adding the above angle we get the whole angle AVB.

Sp. m< AVB = m <AVC + <VCB

m <AVB = 23 + 39

m <AVB = 62

Learn more about Angle here:

https://brainly.com/question/28451077

#SPJ7

Other Questions
what is the rate of change between (29,9) and (33,10)? A scatter plot and a possible line of best fit is shown: Is the line of best fit accurate for the data shown? No, because the line does not touch any points No, because the line should touch every point Yes, because it touches the y-axis Yes, because it passes through the center of the data points How is Argentina ranked among the largest countries of the world? A line in the xy-plane passes through the points (1,-2) And has a slope of 1/3. Which of the following points lies on the line?A) (3,-2)B) (2,-4/3)C) (0,-2)D) (-1,-8/3) What is the mechanical advantage of a pulley with 3 support ropes? A large container of water is sitting on a balance. The weight of the container plus the water is 10 newtons. An object the weight of 7 newtons hanging from a forcemeter is lowered towards the surface of the water . what reading on the force meter would you expect? Researchers are interested in determining whether more women than men prefer the beach to the mountains. In a random sample of 200 women, 45% prefer the beach, whereas in a random sample of 300 men, 52% prefer the beach. What is the 99% confidence interval estimate for the difference between the percentages of women and men who prefer the beach over the mountains? Plz help asap, shdhhshdhshxh g Your savings account earns 1.72% interest. You have $3,000 left over from an internship that you will put into the saving account. There is currently no money in the account :( How many years is it until there is $3,756 in the account? Explain why paying money to the government officials to get water is an undemocratic method.(Point stealers will be reported so better dont do it.) ASAP! GIVING BRAINLIEST! Please read the question THEN answer CORRECTLY! NO guessing. I say no guessing because people usually guess on my questions. What is the value of 1 over 2 x+3.4y when x=3 and y = 4? Which of the following tests include 2 components of health-related fitness when tested? a Mile-run. b PACER c Curl-up. d Push-up. e Planks. f Trunk-lift. Piensa en una historia que quieras escribir. Primero, plantea una virtud o un defecto humano que quieras resaltar. ---[recuerda que las fabulas contienen un mensaje morarilizante what is a single word answer to say _I am_ in spanish? 1The use of a wholesale warehouse in the chain of distribution [QAenables direct selling from manufacturer to retailer.B.enables production to be ahead of demand.C. reduces the chain of distribution.Direduces the cost of goods to consumers. Which of the following parts of ruminant stomach is called the honeycomb? And why?A. Rumen B. ReticulumC. AbomasumD. Omasum which choice is equivalent to the quotient shown here when x>0 Geese Company utilizes the dollar-value LIFO retail inventory method. Its cost-to-retail percentage is 60% based on beginning inventory and 64% based on current-period purchases. The company determined that beginning inventory at retail was $200,000 and that ending inventory at current-year retail prices was $250,000. The current-year price index is 1.10. Rounding to the nearest dollar, ending inventory at base-year retail prices would be _______. In converting 10 pounds to ounces, what unit (omit the number) would youplace in the numerator of your ratio? Use the plural form in your answer.Remember that there are 16 ounces in 1 pound.